LSAT and Law School Admissions Forum

Get expert LSAT preparation and law school admissions advice from PowerScore Test Preparation.

User avatar
 Dave Killoran
PowerScore Staff
  • PowerScore Staff
  • Posts: 5853
  • Joined: Mar 25, 2011
|
#46269
Complete Question Explanation
(The complete setup for this game can be found here: lsat/viewtopic.php?t=14920)

The correct answer choice is (C)

If F and M are on the same team, then from the first and third rules that team will not include R, nor will it include S. The FM team still needs a linguist, however, and this would have to be either N or O. Additionally, since each team must include an anthropologist, whichever team does not include the FM block must include J, the only remaining anthropologist. Because placing J on team 1 results in R and F being included on team 2, we can determine that J cannot be on team 1 in this question. Thus, J must be on team 2, and the FM block must be on team 1.

Only one of the answer choices could be true, and the others should be ruled out:

Answer choice (A) is incorrect because J cannot be on team 1. If J is on team 1, then R must be on team 2 (based on the last rule of the game). Since the FM block cannot be teammates with R, they would have to go to team 1, and then team 1 would have no linguists, and team 2 would have no anthropologists.

Answer choice (B) is incorrect for a similar reason. If R is on team 1, J cannot be on team 1 based on the game’s last rule. Because of the fourth rule, the FM block also must avoid R, leaving no anthropologists available for team 1.

Answer choice (C) is the correct answer choice.

Answer choices (D) and (E) are both incorrect because if there is an NO block, it must be on the team that does not include the FM block (each team has only three spaces available, and the two blocks would combine to produce four researchers). The FM team still needs a linguist, but there is none that is allowable—N and O must be on the other team, and the global rules prohibit R and S from being on the FM team.
User avatar
 lsatstudy2023
  • Posts: 13
  • Joined: Jun 28, 2023
|
#102733
This question was a bit confusing to me; "If F and M" are on the same thing which of the following could be true; Does this mean the F and M can be in either team 1 or team 2?

Your help is greatly appreciated!
 Adam Tyson
PowerScore Staff
  • PowerScore Staff
  • Posts: 5153
  • Joined: Apr 14, 2011
|
#102774
The way the question is asked, lsatstudy2023, does not tell us which team they are both on, so we might consider them on either team. But as Dave explained, it turns out that they cannot both be on Team 2, so we end up making the inference that if they are together, they must be on Team 1, along with either N or O. J must be on Team 2 with a pair of Linguists, and that pair cannot be N and O, although it might include one or the other of them.

Get the most out of your LSAT Prep Plus subscription.

Analyze and track your performance with our Testing and Analytics Package.